a test has 10 multiple-choice questions with 6 choices each, followed by 35 true/false questions. if a student guesses on each equation, how many ways can he answer the questions on the test

Answers

Answer 1

Answer:

6¹⁰×2³⁵

Step-by-step explanation:

he has 6 choices for the first multiple choice question.

and for each of those he had again 6 more choices to answer the second question. 6×6 = 36

so, for all 10 multiple choice questions he answer in

6¹⁰ different ways = 60466176 ways

then there are 35 true/false questions, which are Bausch again multiple choice questions but with only 2 options instead of 6.

so we get 2³⁵ different possibilities. a huge number.

and they're possible for each of the 60466176 ways of the multiple choice part.

so, in total we have

6¹⁰×2³⁵ different answer possibilities.


Related Questions

Help!!!!!!! Thank you!!!!!!!

Answers

Answer:

D

Step-by-step explanation:

The ratio of yellow paint to blue paint is 4:3. We can make the largest amount of green paint by using all of the 20 quarts of yellow paint so we have to solve for x in 4:3 = 20:x, since 4 * 5 = 20, 3 * 5 = x so we use 15 qts of blue paint, therefore we will have 20 + 15 = 35 qts of green paint.

Answer:

D

Step-by-step explanation:

A rhombus has an area of 5 square meters and a side length of 3 meters. In another similar rhombus, the length of a side is 9 meters. What is the area of the second rhombus?
(A) 30 square meters
(B) 45 square meters
(C) 60 square meters
(D) 75 square meters

Answers

Hence the area of the second rhombus is 45 square meters

The area of a rhombus is expressed as

A = base * height

For the rhombus with an area of 5 square meters and a side length of 3 meters

Height = Area/length

Height = 5/3 metres

Since the length of a similar rhombus is 9meters, the scale factor will be expressed as;

k = ratio of the lengths = 9/3

k = 3

Height of the second rhombus = 3 * height of the first rhombus

Height of the second rhombus = 3 * 5/3

Height of the second rhombus = 5 meters

Area of the second rhombus = length * height

Area of the second rhombus = 5 * 9

Area of the second rhombus = 45 square meters

Hence the area of the second rhombus is 45 square meters

Learn more here: brainly.com/question/20247331

The correct option is option B;

(B) 45 square meters

The known parameters in the question are;

The area of the rhombus, A₁ = 5 m²

The length of one of the sides of the rhombus, a = 3 m

The length of a side in a similar rhombus, b = 9 m

The unknown parameter;

The area of the second rhombus

Strategy or method;

We have that two shapes are similar if their corresponding sides are proportional

From the above statement we get that the ratio of the areas of the two shapes is equal to the square of the ratio of the lengths of the corresponding sides of the two shapes of follows;

[tex]\begin{array}{ccc}Length \ Ratio&&Area \ Ratio\\\dfrac{a}{b} &&\left (\dfrac{a}{b} \right)^2 \\&&\end{array}[/tex]

Let the area of the second rhombus be A₂, we get;

[tex]Area \ ratio = \dfrac{A_1}{A_2} = \left( \dfrac{a}{b} \right)^2[/tex]

Where;

a = 3 m, b = 9 m, and A₁ = 5 m², we get;

[tex]Area \ ratio = \dfrac{5 \ m^2}{A_2} = \left( \dfrac{3 \, m}{9 \, m} \right)^2 = \dfrac{1}{9}[/tex]

Therefore;

9 × 5 m² = A₂ × 1

A₂ = 45 m²

The area of the second rhombus, A₂ = 5 m².

Learn more about scale factors here;

https://brainly.com/question/20247331

What is the equation of the line of best fit for the following data? Round the
slope and y-intercept of the line to three decimal places.

Answers

Answer:

the line of best fit can be approximated to:

y = -1.560 x + 22.105

Step-by-step explanation:

You are most likely expected to use a graphing tool are statistical program to calculate this. So enter the list of x-values separate from the list of y values and run the tool in linear regression mode.

Look at the attached image with the actual results including the line of best fit.

The equation can be written (rounding slope and y-intercept to 3 decimals) as:

y = -1.560 x + 22.105

Which number is divisible by 5? 99 45 83 94

Answers

Answer:

45

Step-by-step explanation:

because 5•9=45 so yeah that's the answer

Tina's age is 4 years less than 3 times her niece's age. If her niece's age is x years, which of the following expressions best shows Tina's age? x − 4 4x − 3 3x − 4 4 − 3x

Answers

Answer:

3x - 4

Step-by-step explanation:

As Tina's age is 3 into x ( 3 x x= 3x)but 4years less (-4)

Therefore Tina's age is 3x - 4

Answer:

3x - 4

Step-by-step explanation:

Use these representations:  niece's age: x

We triple x and then subract 4 years from the result, obtaining:

Tina's age:  3x - 4

The mean number of rushing yards for one NFL team was less than 99 yards per game. If a hypothesis test is performed, how should you interpret a decision that rejects the null hypothesis?

Answers

Question options :

A. There is sufficient evidence to reject the claim

u < 99.

B. There is sufficient evidence to support the claim

u < 99.

C. There is not sufficient evidence to reject the claim

u < 99.

D. There is not sufficient evidence to support the claim

u< 99.

Answer:

B. There is sufficient evidence to support the claim

u < 99.

Step-by-step explanation:

We construct the n*ll and alternative hypotheses to support our claim

The n*ll hypothesis :H0

The alternative hypothesis : Ha

N*ll hypothesis =H0: u=99

Alternative hypothesis =Ha: u<99

So if n*ll hypothesis (H0) u=99 is rejected, then we accept the alternative hypothesis that u<99

we can therefore have sufficient evidence to support our claim that u<99

Given: 8(y + 2) = 48
Solve for “y.”

16
-6
20
4

Answers

To equal 48 y should equal 4

Since (y+2) is in parenthesis it always goes first

Substitute 4 for y so added to 2 it equals 6

6x8=48

The value of y will be equal to 4. The correct option is D.

What is an expression?

Expression in maths is defined as the collection of the numbers variables and functions by using signs like addition, subtraction, multiplication, and division.

Numbers (constants), variables, operations, functions, brackets, punctuation, and grouping can all be represented by mathematical symbols, which can also be used to indicate the logical syntax's order of operations and other features.

The given expression 8(y + 2) = 48 will be solved for y as below:-

8(y + 2) = 48

Divide both sides by 8 and solve.

[ 8 (y + 2) ] / 8 = 48 / 8

y + 2 = 6

Substract 2 from both the sides to get the value of y.

y + 2 - 2 = 6 -2

y = 4

Therefore, the value of y will be equal to 4. The correct option is D.

To know more about Expression follow

https://brainly.com/question/723406

#SPJ5

The sum of two numbers is 49 and the difference between these two numbers is 9. What are these two numbers?​Shown working out please

Answers

Let numbers be x and y

ATQ

x+y=49---(1)

x-y=9---(2)

Adding both

[tex]\\ \sf\longmapsto 2x=58[/tex]

[tex]\\ \sf\longmapsto x=\dfrac{58}{2}[/tex]

[tex]\\ \sf\longmapsto x=29[/tex]

Now putting value in eq(2)

[tex]\\ \sf\longmapsto x-y=9[/tex]

[tex]\\ \sf\longmapsto 29-9=y[/tex]

[tex]\\ \sf\longmapsto y=20[/tex]

Graph: y < 3x + 1 please help me

Answers

Answer:

Using a graphing calc.

Step-by-step explanation:

What is tan 30°?
60
2
1
90°
30"
V3
O A.
B. 1
O c. 2
O D. 7/ 룸
O E
1 / 3
Eg
O E

Answers

Answer:

Hello,

What is tan 30°?

[tex]tan(30^o)=\dfrac {\sqrt{3} }{3}[/tex]

Step-by-step explanation:

[tex]sin(30^o)=\dfrac{1}{2} \\\\cos(30^o)=\dfrac{\sqrt{3} }{2} \\\\\\tan(30^o)=\dfrac{sin(30^o)}{cos(30^o)} \\tan(30^o)=\dfrac{\dfrac{1}{2} } { \dfrac{\sqrt{3} }{2} }\\\\ =\dfrac {1*2}{2*\sqrt{3} }\\\\ =\dfrac {\sqrt{3} }{3}[/tex]

The value of tan 30° is 1/√3

What is tangent of an angle?

The tangent of an angle is the ratio of the length of the opposite side to the length of the adjacent side.

In other words, it is the ratio of sine and cosine function of an acute angle such that the value of cosine function should not equal to zero.

Tan 30° = sin 30° / cos 30°

We know that, sin 30° = 1/2

cos 30° = √3/2

Therefore,

Tan 30° = 1/2 ÷ √3/2

Tan 30° = 1/2 x 2/√3

Tan 30° = 1/√3

Hence, the value of tan 30° is 1/√3

Learn more about tangent of an angle, click;

https://brainly.com/question/10053881

#SPJ7

What is the error in this problem

Answers

Answer:

12). LM = 37.1 units

13). c = 4.6 mi

Step-by-step explanation:

12). LM² = 23² + 20² - 2(23)(20)cos(119)°

    LM² = 529 + 400 - 920cos(119)°

    LM² = 929 - 920cos(119)°

    LM = [tex]\sqrt{929+446.03}[/tex]

          = [tex]\sqrt{1375.03}[/tex]

          = 37.08

          ≈ 37.1 units

13). c² = 5.4² + 3.6² - 2(5.4)(3.6)cos(58)°

    c² = 29.16 + 12.96 - 38.88cos(58)°

    c² = 42.12 - 38.88cos(58)°

    c = [tex]\sqrt{42.12-20.603}[/tex]

    c = [tex]\sqrt{21.517}[/tex]

    c = 4.6386

    c ≈ 4.6 mi

The Bay Area Online Institute (BAOI) has set a guideline of 60 hours for the time it should take to complete an independent study course. To see if the guideline needs to be changed and if the actual time taken to complete the course exceeds60 hours, 16 students are randomly chosen and the average time to complete the course was 68hours with a standard deviation of 20 hours. What inference can BAOI make about the time it takes to complete this course?

Answers

Answer:

 At the 5% level, BAOI can infer that the average time to complete does not exceeds 60 hours.

Step-by-step explanation:

From the question we are told that

   The  population mean is [tex]\mu = 60 \ hr[/tex]

    The sample size is  [tex]n = 16[/tex]

    The  sample mean is  [tex]\= x = 68 \ hr[/tex]

     The  standard deviation is  [tex]\sigma = 20 \ hr[/tex]

The  null hypothesis is  [tex]H_o : \mu = 60[/tex]

The  alternative [tex]H_a : \mu > 60[/tex]

Here we would assume the level of significance of this test to be  

         [tex]\alpha = 5\% = 0.05[/tex]

Next we will obtain the critical value of the level of significance from the normal distribution table, the value is    [tex]Z_{0.05} = 1.645[/tex]

  Generally the test statistics  is mathematically represented as

           [tex]t = \frac{ \= x - \mu}{ \frac{ \sigma }{\sqrt{n} } }[/tex]

substituting values

           [tex]t = \frac{ 68 - 60 }{ \frac{ 20 }{\sqrt{16} } }[/tex]

          [tex]t = 1.6[/tex]

Looking at the value of t and  [tex]Z_{\alpha }[/tex] we see that [tex]t< Z_{\alpha }[/tex] hence we fail to reject the null hypothesis

   This means that there no sufficient evidence to conclude that it takes more than 60 hours to complete the course

So

   At the 5% level, BAOI can infer that the average time to complete does not exceeds 60 hours.

I need help with the answer

Answers

Answer:

Option B, x ≈ -2.25

Step-by-step explanation:

3^x-2=(x-1)/(x^2+x-1)

or x ≈ -2.21166

so it's closest to the answer of the 2nd option

In an opinion poll, 29% of 100 people sampled said they were strongly opposed to the state lottery. What is the approximate standard error of the sample proportion?

Answers

Answer:

  The approximate standard error of the sample proportion is [tex]SE = 0.0454[/tex]

Step-by-step explanation:

From the question we are told that

      The  sample proportion is  [tex]\r p = 0.29[/tex]

       The  sample size is  n =  100

     

Generally the standard error of the sample proportion is mathematically represented as

         [tex]SE = \sqrt{ \frac{ \r p (1- \r p )}{n} }[/tex]

substituting values    

         [tex]SE = \sqrt{ \frac{ 0.29 (1- 0.29 )}{ 100 } }[/tex]

         [tex]SE = 0.0454[/tex]

Simplify.
10
3
(2.8 +1.2)
6
16
18.4
20
26

Answers

Answer:

see similar to the number of the number

A basketball player scored 33 points during a game by shooting 1-point free throws, 2-point field goals, and 3-point field goals. The player scored 17 times. She scored 3 more 2-point field goals than 1-point free throws. The system of equations below represents the situation, where x is the number of 1-point free throws, y is the number of 2-point field goals, and z is the number of 3-point field goals. x + y + z = 17 x + 2y + 3z = 33 y – x = 3

Answers

Answer:

No. of 1 pt free throws = 5, No. of 2 pt goals = 8, No. of 3 pt goals = 4

Step-by-step explanation:

Equations : x + y + z = 17 [ Total times taken to score ]

1x + 2y + 3z = 33 [ Total Score ]

Also, y = x + 3

Putting the value of 'y' in both equations :

x + (x + 3)+ z = 17 → 2x + 3 + z = 17 → 2x + z = 14  (i)

1x + 2 (x + 3) + 3z = 33 →  x + 2x + 6 + 3z = 33 → 3x + 3z = 27 (ii)

Solving these equations :

From (i), z = 14 - 2x

Putting this value in (ii), 3x + 3(14 - 2x) = 27 → 3x + 42 - 6x = 27

42 - 3x = 27 → 3x = 15 → x = 5

y = x + 3 = 5 + 3 → y = 8

z = 17 - x - y → z = 17 - 5 - 8 = 17 - 13 → z = 4

Answer:

4

Step-by-step explanation:

The digits 0,1,2,3,4,5 and 6 are used to make 3 digit codes

In case where digits may be repeated, how many codes are numbers that are greater than 300 and exactly divisible by 5?​

Answers

Answer:

345/5=69

Step-by-step explanation:

345/5=69

355/5=71

83=4k-7(1+7k) How to solve

Answers

Answer:

k = -2

Step-by-step explanation:

83=4k-7(1+7k)

Distribute

83=4k-7-49k

Combine like terms

83 = -45k -7

Add 7 to each side

83+7 = -45k-7+7

90 = -45k

Divide each side by -45

90/-45 = -45k/-45

-2 = k

Answer:

k = -2

Step-by-step explanation:

Step 1: Use 7 to open the bracket :

-7(1+7k)=-7-49k

Step 2: Collect like terms

Step 3 : Divide both sides of the equation by -45

[tex]83=4k-7(1+7k) \\ \\83 = 4k-7-49k\\\\ 83+7=4k-49k\\\\90 = -45k\\\\\frac{90}{-45} = \frac{-45k}{-45} \\\\k = -2[/tex]

PLEASE HELP!!!!!!! FIRST CORRECT ANSWER WILL BE THE BRAINLIEST....PLEASE HELP
Lunch Choices of Students
The bar graph shows the percent of students that chose each food in the school
cafeteria. Which statement about the graph is true?

Answers

Answer:

(2) If 300 lunches were sold, then 120 chose tacos.

Step-by-step explanation:

We can evaluate each option and see if it makes it true.

For 1: If 200 lunches were served, 10 more students chose pizza over hotdogs.

We can find how many pizzas/hotdogs were given if 200 lunches were served by relating it to 100.

20% chose hotdog, which is [tex]\frac{20}{100}[/tex]. Multiply both the numerator and denominator by two: [tex]\frac{40}{200}[/tex] - so 40 students chose hotdogs.

Same logic for pizza: 30% chose pizza - [tex]\frac{30}{100} = \frac{60}{200}[/tex] so 60.

60 - 40 = 20, not 10, so 1 doesn't work.

2: If 300 lunches were sold, then 120 chose tacos.

Let's set up a proportion again. 40% of 100 is 40.

[tex]\frac{40}{100} = \frac{40\cdot3}{300} = \frac{120}{300}[/tex]

So 120 tacos were chosen - yes this works!

Hope this helped!

What are the solution(s) of the quadratic equation 98 - x2 = 0?
x = +27
Ox= +63
x = +7/2
no real solution

Answers

Answer:

±7 sqrt(2) = x

Step-by-step explanation:

98 - x^2 = 0

Add x^2 to each side

98 =x^2

Take the square root of each side

±sqrt(98) = sqrt(x^2)

±sqrt(49*2) = x

±7 sqrt(2) = x

Answer:

[tex]\huge \boxed{{x = \pm 7\sqrt{2} }}[/tex]

Step-by-step explanation:

[tex]98-x^2 =0[/tex]

[tex]\sf Add \ x^2 \ to \ both \ sides.[/tex]

[tex]98=x^2[/tex]

[tex]\sf Take \ the \ square \ root \ of \ both \ sides.[/tex]

[tex]\pm \sqrt{98} =x[/tex]

[tex]\sf Simplify \ radical.[/tex]

[tex]\pm \sqrt{49} \sqrt{2} =x[/tex]

[tex]\pm 7\sqrt{2} =x[/tex]

[tex]\sf Switch \ sides.[/tex]

[tex]x= \pm 7\sqrt{2}[/tex]

Ellen is making jewelry sets that contain a bracelet and a pair of earrings. Each bracelet uses 3 times as many beads as one earring. Each bracelet uses 3 as times as many beads as one earring . Ellen uses 13 beads for each earring. How many beads does Ellen need to make one jewelry set?

Answers

So there are a pair of earrings and a Bracelet.

It's given that the Bracelet uses 3 times the number of beads that's used in making a single earring.

It's also given that one single earing has 13 beads. So a single bracelet would have (3×13) beads .... and that's equal to 39.

Making a single set of jewellery needs a pair of earrings and a Bracelet.

So total number of required beads will be =

39 + 13 + 13 = 65

Angelique travels 75 miles. Jamila travels 115 kilometres. Show that Angelique has travelled futher than Jamila.

Answers

Answer:

Step-by-step explanation:

-we know :

we must have the same unit on the distance traveled in order to be able to compare distances  

1 mile = 1.6 kilometers

-Angelique traveled 75 miles ( 120 kilometers)

75 miles = 1.6 *75 = 120 kilometers

-Jamila traveled 115 kilometers  is given in the problem

-Angelique has travelled further than Jamila because

120 kilometers > 115 kilometers

A machine that produces ball bearings has initially been set so that the true average diameter of the bearings it produces is 0.500 in. A bearing is acceptable if its diameter is within 0.004 in. of this target value. Suppose, however, that the setting has changed during the course of production, so that the bearings have normally distributed diameters with a mean 0.499 in. and standard deviation 0.002 in. What percentage of bearings will now not be acceptable

Answers

Answer:

the percentage of  bearings   that will  not be acceptable = 7.3%

Step-by-step explanation:

Given that:

Mean = 0.499

standard deviation = 0.002

if the true average diameter of the bearings it produces is 0.500 in and bearing is acceptable if its diameter is within 0.004 in.

Then the ball bearing acceptable range = (0.500 - 0.004, 0.500 + 0.004 )

= ( 0.496 , 0.504)

If x represents the diameter of the bearing , then the probability for the  z value for the random variable x with a mean and standard deviation can be computed as follows:

[tex]P(0.496\leq X \leq 0.504) = (\dfrac{0.496 - \mu}{\sigma} \leq \dfrac{X -\mu}{\sigma} \leq \dfrac{0.504 - \mu}{\sigma})[/tex]

[tex]P(0.496\leq X \leq 0.504) = (\dfrac{0.496 - 0.499}{0.002} \leq \dfrac{X -0.499}{0.002} \leq \dfrac{0.504 - 0.499}{0.002})[/tex]

[tex]P(0.496\leq X \leq 0.504) = (\dfrac{-0.003}{0.002} \leq Z \leq \dfrac{0.005}{0.002})[/tex]

[tex]P(0.496\leq X \leq 0.504) = (-1.5 \leq Z \leq 2.5)[/tex]

[tex]P(0.496\leq X \leq 0.504) = P (-1.5 \leq Z \leq 2.5)[/tex]

[tex]P(0.496\leq X \leq 0.504) = P(Z \leq 2.5) - P(Z \leq -1.5)[/tex]

From the standard normal tables

[tex]P(0.496\leq X \leq 0.504) = 0.9938-0.0668[/tex]

[tex]P(0.496\leq X \leq 0.504) = 0.927[/tex]

By applying the concept of probability of a  complement , the percentage of bearings will now not be acceptable

P(not be acceptable)  = 1 - P(acceptable)

P(not be acceptable)  = 1 - 0.927

P(not be acceptable)  = 0.073

Thus, the percentage of  bearings   that will  not be acceptable = 7.3%

Find the counterclockwise circulation and outward flux of the field F=7xyi+5y^2j around and over the boundary of the region C enclosed by the curves y=x^2 and y=x in the first quadrant.

Answers

Split up the boundary of C (which I denote ∂C throughout) into the parabolic segment from (1, 1) to (0, 0) (the part corresponding to y = x ²), and the line segment from (1, 1) to (0, 0) (the part of ∂C on the line y = x).

Parameterize these pieces respectively by

r(t) = x(t) i + y(t) j = t i + t ² j

and

s(t) = x(t) i + y(t) j = (1 - t ) i + (1 - t ) j

both with 0 ≤ t ≤ 1.

The circulation of F around ∂C is given by the line integral with respect to arc length,

[tex]\displaystyle \int_{\partial C}\mathbf F\cdot\mathbf T \,\mathrm ds[/tex]

where T denotes the tangent vector to ∂C. Split up the integral over each piece of ∂C :

• on the parabolic segment, we have

T = dr/dt = i + 2t j

• on the line segment,

T = ds/dt = -i - j

Then the circulation is

[tex]\displaystyle \int_{\partial C}\mathbf F\cdot\mathbf T\,\mathrm ds = \int_0^1 (7t^3\,\mathbf i+5t^4\,\mathbf j)\cdot(\mathbf i+2t\,\mathbf j)\,\mathrm dt + \int_0^1 (7(1-t)^2\,\mathbf i+5(1-t)^2\,\mathbf j)\cdot(-\mathbf i-\mathbf j)\,\mathrm dt \\\\ = \int_0^1 (7t^3+10t^5)\,\mathrm dt - 12 \int_0^1 (1-t)^2\,\mathrm dt =\boxed{-\frac7{12}}[/tex]

Alternatively, we can use Green's theorem to compute the circulation, as

[tex]\displaystyle\int_{\partial C}\mathbf F\cdot\mathbf T\,\mathrm ds = \iint_C\frac{\partial(5y^2)}{\partial x} - \frac{\partial(7xy)}{\partial y}\,\mathrm dx\,\mathrm dy \\\\ = -7\int_0^1\int_{x^2}^x x\,\mathrm dx \\\\ = -7\int_0^1 xy\bigg|_{y=x^2}^{y=x}\,\mathrm dx \\\\ =-7\int_0^1(x^2-x^3)\,\mathrm dx = -\frac7{12}[/tex]

The flux of F across ∂C is

[tex]\displaystyle \int_{\partial C}\mathbf F\cdot\mathbf N \,\mathrm ds[/tex]

where N is the normal vector to ∂C. While T = x'(t) i + y'(t) j, the normal vector is N = y'(t) i - x'(t) j.

• on the parabolic segment,

N = 2t i - j

• on the line segment,

N = - i + j

So the flux is

[tex]\displaystyle \int_{\partial C}\mathbf F\cdot\mathbf N\,\mathrm ds = \int_0^1 (7t^3\,\mathbf i+5t^4\,\mathbf j)\cdot(2t\,\mathbf i-\mathbf j)\,\mathrm dt + \int_0^1 (7(1-t)^2\,\mathbf i+5(1-t)^2\,\mathbf j)\cdot(-\mathbf i+\mathbf j)\,\mathrm dt \\\\ = \int_0^1 (14t^4-5t^4)\,\mathrm dt - 2 \int_0^1 (1-t)^2\,\mathrm dt =\boxed{\frac{17}{15}}[/tex]

​Steven’s basketball team won 5 out of their first 12 games. If they continue to win at this​ rate, how many games will they lose if they play 49​ games?

Answers

Answer:

29 losses in 49 games.

Step-by-step explanation:

The number of games that they lost in the first 12 = 12 - 5 = 7

7/12 = x / 49           Multiply both sides by 49

7*49 / 12 = x

x = 343/12

x = 28.5833

What do you do with the 0.5833? I would say round up to 29.

Find the solutions of x^2+30 = 0

please give detailed steps!

Answers

Answer:

x= i√30

Step-by-step explanation:

I'm going to go into this under the assumption that you've covered imaginary numbers based on the question. If I'm wrong then sorry about that.

Okay, so first you want to subtract 30 from both sides

x^2=-30

Then you take the square root of each side.

√(x^2)=√-30

x=√-30

Since it's impossible to square a number to get a negative number, you'll end up with an imaginary number. You have to rewrite x=-30 to get rid of the negative sign under the radical. Rewriting this will also indicate that it's an imaginary number.

Final answer: x = i√30

If the lengths of the legs of a right triangle are 3 and 5, what is the length of the hypotenuse?

Answers

P=5B=3

Using Pythagorean theorem

[tex]\\ \sf\longmapsto H^2=P^2+B^2[/tex]

[tex]\\ \sf\longmapsto H^2=5^3+3^2[/tex]

[tex]\\ \sf\longmapsto H^2=25+9[/tex]

[tex]\\ \sf\longmapsto H^2=34[/tex]

[tex]\\ \sf\longmapsto H=\sqrt{34}[/tex]

[tex]\\ \sf\longmapsto H=5.22[/tex]

What is 5 over 30= 3 over c

Answers

Answer:

c=18

Step-by-step explanation:

5/30=3/c

1/6=3/18

1✖️3=3

6✖️3=18

A log of wood weighs 120kg. After drying, it now weighs 80kg. Find the moisture content of the wood in percentage.​

Answers

Answer: 33% is moisture content

Step-by-step explanation:

120kg - 80kg = 40kg

40 of 120 is %

Work:

40/120 = 0.33

0.33x100

= 33%

Identify the type of equation: y-6 = 7(x+8)

Answers

Step-by-step explanation:

Recognize the relation between the graph and the slope–intercept form of an equation of a line

Identify the slope and y-intercept form of an equation of a line

Graph a line using its slope and intercept

Choose the most convenient method to graph a line

Graph and interpret applications of slope–intercept

Use slopes to identify parallel lines

Use slopes to identify perpendicular lines

Other Questions
What should be added to p to get p+q? *PLEASE ANSWER TY* A scale factor of 2 is applied to the dimensions of the prism. What effect will this have on the volume? A) Which of the three types of needs currently drive you? Will that have to change once you find a career-oriented job? What's 955 35 ?[tex]955 \div 35[/tex] Given the solution [tex]y_{1}(x)[/tex] from EDO below, develop a second solution. [tex]x\frac{d^{2}y }{dx^{2} } +3\frac{dy}{dx} -y=0,\\y_{1} (x)=1+\frac{x}{3} +\frac{x^{2} }{24} +\frac{x^{3} }{360} + ...[/tex] Find the length of GV A. 43.92 B. 33.1 C. 41.45 D. 68.87 Before working at the hospital, Beth was given a Mantoux skin test to detect tuberculosis. If it were positive, the site of the test would become hardened and red. What type of response is this? Once the second world war was over, fashion magazines ceased featuring the work of American designers and returned to focus only on the work of Parisian designers. Group of answer choices 13. If atoms from two different elements react to form a compound, the element with a highernumberwill have a negative oxidationA. atomic radiusB. energyC. electronegativityD. atomic number Which are the two most popular candidates for gamma-ray bursters? Group of answer choices collisions between a white dwarf and a giant, and merger of two neutron stars hypernova making a black hole, and merger of two neutron stars formation of uranium in the core of a supergiant, and collisions of white dwarfs mergers of two black holes, and merger of a neutron star and a white dwarf hypernova making pulsars, and mergers of two white dwarfs what is the similarity between ordinary and special resolution 2. El sentido contextual de la palabra SUBSUMIR esA) Abarcar.B) Mediar.C) Potenciar.D) Describir.E) Demostrar. The recognition of the need for organizations to improve the state of people, the planet, and profit simultaneously if they are to achieve sustainable, long-term growth (Connect Chapter 4) is referred to as What is the symbolic meaning of the relationship between light and dark in the story in the Araby 6. The skin on the leg is___________to the muscle in the leg distinguish between fixed cost and Variable Cost Segn la lectura, al contrario que la literatura del Siglo de Oro, el Answer estaba ms preocupado por la vida real. (1 point) Will give brainliestJillians school is selling tickets for a play. The tickets cost $10.50 for adults and $3.75 for students. The ticket sales for opening night totaled $2071.50. The equation 10.50 a plus 3.75 b equals 2071.50, where a is the number of adult tickets sold and b is the number of student tickets sold, can be used to find the number of adult and student tickets. If 82 students attended, how may adult tickets were sold? ___adult tickets A bond issue with a face amount of $1,200,000 bears interest at the rate of 9%. The current market rate of interest is 10%. These bonds will sell at a price that is: Enrique has to pay 80% of a $200 phone bill. Explain how to use equivalent ratios to find 80% of $200. please help